Divisibility rules and proof by contradiction

Click For Summary
The discussion centers on proving that for integers x, y, and z with no common divisor, the condition involving divisibility by 5 leads to a contradiction regarding their greatest common divisor. The poster is attempting to show that if 5 divides one of the terms, it must divide all three integers. They encounter difficulties when assuming 5 divides a specific expression involving squares of integers. A suggestion is made to analyze the square terms modulo 5 to find an expression that must be divisible by 5. This approach aims to demonstrate that at least one of the square terms must be divisible by 5, aiding in the proof.
tt2348
Messages
141
Reaction score
0
I posted this in the number theory forum to no success... so I figured maybe the homework help people would have some input

Let x,y,z be integers with no common divisor satisfying a specific condition, which boils down to
5|(x+y-z) and 2*5^{4}k=(x+y)(z-y)(z-x)((x+y)^2+(z-y)^2+(z-x)^2)
or equivalently 5^{4}k=(x+y)(z-y)(z-x)((x+y-z)^2-xy+xz+yz)
I want to show that GCD(x,y,z)≠1, starting with the assumption 5 dividing (x+y), (z-y), or (z-x) results in x,y or z being divisible by 5. then it's easy to show that 5 divides another term, implying 5 divides all three.
I run into trouble assuming 5 divides the latter part, 2((x+y)^2+(z-y)^2+(z-x)^2)=((x+y-z)^2-xy+xz+yz) and showing the contradiction from that point.
Any hints?
 
Physics news on Phys.org
Hello tt2348,
Leave the square terms unchanged and consider all possible values of the square of any integer mod 5 .There are three squares here() .How will you get them to get an expression divisible by 5 ?There lies the answer that one of the square terms has to be divisible by 5.
Hoping this helps.
regards
Yukoel
 
Question: A clock's minute hand has length 4 and its hour hand has length 3. What is the distance between the tips at the moment when it is increasing most rapidly?(Putnam Exam Question) Answer: Making assumption that both the hands moves at constant angular velocities, the answer is ## \sqrt{7} .## But don't you think this assumption is somewhat doubtful and wrong?

Similar threads

Replies
1
Views
1K
  • · Replies 8 ·
Replies
8
Views
2K
  • · Replies 4 ·
Replies
4
Views
2K
Replies
45
Views
5K
  • · Replies 5 ·
Replies
5
Views
2K
  • · Replies 3 ·
Replies
3
Views
1K
Replies
3
Views
2K
  • · Replies 6 ·
Replies
6
Views
2K
  • · Replies 3 ·
Replies
3
Views
2K
Replies
2
Views
1K